Difference between revisions of "2019 AMC 10B Problems/Problem 1"
(→Solution 1) |
m (changed headers) |
||
Line 7: | Line 7: | ||
<math>\textbf{(A) } \frac{5}{8} \qquad \textbf{(B) } \frac{4}{5} \qquad \textbf{(C) } \frac{7}{8} \qquad \textbf{(D) } \frac{9}{10} \qquad \textbf{(E) } \frac{11}{12}</math> | <math>\textbf{(A) } \frac{5}{8} \qquad \textbf{(B) } \frac{4}{5} \qquad \textbf{(C) } \frac{7}{8} \qquad \textbf{(D) } \frac{9}{10} \qquad \textbf{(E) } \frac{11}{12}</math> | ||
− | ==Solution 1== | + | ==Solution== |
+ | ===Solution 1=== | ||
Let the first jar's volume be <math>A</math> and the second's be <math>B</math>. It is given that <math>\frac{3}{4}A=\frac{5}{6}B</math>. We find that <math>\frac{A}{B}=\frac{\left(\frac{3}{4}\right)}{\left(\frac{5}{6}\right)}=\boxed{\textbf{(D) }\frac{9}{10}}.</math> | Let the first jar's volume be <math>A</math> and the second's be <math>B</math>. It is given that <math>\frac{3}{4}A=\frac{5}{6}B</math>. We find that <math>\frac{A}{B}=\frac{\left(\frac{3}{4}\right)}{\left(\frac{5}{6}\right)}=\boxed{\textbf{(D) }\frac{9}{10}}.</math> | ||
Line 13: | Line 14: | ||
We already know that this is the ratio of the smaller to the larger volume because it is less than <math>1.</math> | We already know that this is the ratio of the smaller to the larger volume because it is less than <math>1.</math> | ||
− | ==Solution 2== | + | ===Solution 2=== |
An alternate solution is to plug in some maximum volume for the first container - let's say <math>72</math>, so there was a volume of <math>60</math> in the first container, and then the second container also has a volume of <math>60</math>, so you get <math>60 \cdot \frac{4}{3} = 80</math>. Thus the answer is <math>\frac{72}{80} = \boxed{\textbf{(D) }\frac{9}{10}}</math>. | An alternate solution is to plug in some maximum volume for the first container - let's say <math>72</math>, so there was a volume of <math>60</math> in the first container, and then the second container also has a volume of <math>60</math>, so you get <math>60 \cdot \frac{4}{3} = 80</math>. Thus the answer is <math>\frac{72}{80} = \boxed{\textbf{(D) }\frac{9}{10}}</math>. |
Revision as of 19:15, 1 February 2020
- The following problem is from both the 2019 AMC 10B #1 and 2019 AMC 12B #1, so both problems redirect to this page.
Problem
Alicia had two containers. The first was full of water and the second was empty. She poured all the water from the first container into the second container, at which point the second container was full of water. What is the ratio of the volume of the first container to the volume of the second container?
Solution
Solution 1
Let the first jar's volume be and the second's be . It is given that . We find that
We already know that this is the ratio of the smaller to the larger volume because it is less than
Solution 2
An alternate solution is to plug in some maximum volume for the first container - let's say , so there was a volume of in the first container, and then the second container also has a volume of , so you get . Thus the answer is .
~IronicNinja
See Also
2019 AMC 10B (Problems • Answer Key • Resources) | ||
Preceded by First Problem |
Followed by Problem 2 | |
1 • 2 • 3 • 4 • 5 • 6 • 7 • 8 • 9 • 10 • 11 • 12 • 13 • 14 • 15 • 16 • 17 • 18 • 19 • 20 • 21 • 22 • 23 • 24 • 25 | ||
All AMC 10 Problems and Solutions |
2019 AMC 12B (Problems • Answer Key • Resources) | |
Preceded by First Problem |
Followed by Problem 2 |
1 • 2 • 3 • 4 • 5 • 6 • 7 • 8 • 9 • 10 • 11 • 12 • 13 • 14 • 15 • 16 • 17 • 18 • 19 • 20 • 21 • 22 • 23 • 24 • 25 | |
All AMC 12 Problems and Solutions |
The problems on this page are copyrighted by the Mathematical Association of America's American Mathematics Competitions.